IMO 2010 #6
by Wolstenholme, Nov 5, 2014, 8:57 PM
Let
be a sequence of positive real numbers, and
be a positive integer, such that
![\[a_n = \max \{ a_k + a_{n-k} \mid 1 \leq k \leq n-1 \} \ \textrm{ for all } \ n > s.\]](//latex.artofproblemsolving.com/f/1/2/f12c9c5e0f154549118a33a2f359329333f432a7.png)
Prove there exist positive integers
and
, such that
![\[a_n = a_{\ell} + a_{n - \ell} \ \textrm{ for all } \ n \geq N.\]](//latex.artofproblemsolving.com/b/0/b/b0b2bd4507621302eeee27ae50ed9c0afd72c6ef.png)
Proof:
So this took me quite a while to solve, so as I go through the proof I will include the motivation for each step.
The first thing to ask yourself is "how does
grow"? What function
seems to satisfy
Well, clearly the answer is a function of the form
for some constant
This motivates the following lemma:
Lemma 1: Let
such that
for all
Then
for all 
Proof: We proceed by strong induction. The base case when
follows from the definition of
Now for any
we have for some positive integer
that
as desired.
Now, what are we looking for? Well, we want the sequence
to follow a sort of pattern. Knowing its general asymptotics, we are motivated to do the following: let
for all
Note that by Lemma 1 this sequence consists only of nonnegative numbers.
If you've written out some sequences, it should be clear that our
is exactly the
that the problem asks to find. So wouldn't it be nice to rephrase what we want to prove in terms of the new sequence
Well, after some searching, we find that in fact all we want to show is that for sufficiently large
, we have that
for all
Let's write this as a lemma.
Lemma 2: To solve the problem, it suffices to show that for all sufficiently large
we have 
Proof: Note that
which proves the lemma.
So, how can we show this? Well, we know that the sequence
is bounded from below by
so it would suffices to show that the sequence is weakly decreasing; namely, that
for all
Rephrasing this in terms of the original sequence of
's, we find that it suffices to show that
but this follows immediately from the definition of the
's so we are done.


![\[a_n = \max \{ a_k + a_{n-k} \mid 1 \leq k \leq n-1 \} \ \textrm{ for all } \ n > s.\]](http://latex.artofproblemsolving.com/f/1/2/f12c9c5e0f154549118a33a2f359329333f432a7.png)
Prove there exist positive integers


![\[a_n = a_{\ell} + a_{n - \ell} \ \textrm{ for all } \ n \geq N.\]](http://latex.artofproblemsolving.com/b/0/b/b0b2bd4507621302eeee27ae50ed9c0afd72c6ef.png)
Proof:
So this took me quite a while to solve, so as I go through the proof I will include the motivation for each step.
The first thing to ask yourself is "how does





Lemma 1: Let





Proof: We proceed by strong induction. The base case when





Now, what are we looking for? Well, we want the sequence



If you've written out some sequences, it should be clear that our






Lemma 2: To solve the problem, it suffices to show that for all sufficiently large


Proof: Note that

So, how can we show this? Well, we know that the sequence






